Saturday 28 December 2019

UPSC 2019 WEEKLY CURRENT AFFAIR TEST DATE 15 TO 21 DEC. 2019


TECKPKPRAJAPAT 
Image result for UPSC IMAGES
WEEKLY CURRENT AFFAIR TEST
DATE 15 TO 21 DEC. 2019

1.     Consider the following statements regarding Anti-tank guided missile (ATGM).

1.      It is primarily designed to hit and destroy heavily armored military vehicles.
2.      The Indian Army has inducted Israel-made anti-tank guided missiles (ATGM) Spike along the LoC in Jammu and Kashmir.
3.      Known as the ‘fire and forget’ missiles, Spike ATGMs are portable by men.

Which of the above statements is/are correct?

a)     1, 2
b)     1, 3
c)      2, 3
d)     1, 2, 3


2.     India’s Polar Satellite Launch Vehicle, PSLV-C47 has launched Cartosat-3. Cartosat-3 is mainly used for

1.      Weather mapping
2.      Measure wind speeds and direction
3.      Urban planning
4.      Coastal land use

Select the correct answer code:

a)     1, 2, 3
b)     1, 3, 4
c)      2, 3, 4
d)     1, 2, 3, 4


3.     Consider the following statements.

1.      Guru Ghasidas National Park is set to become Chattisgarh’s first tiger reserve.
2.      In-principle approval has to be accorded by the National Tiger Conservation Authority for the creation of new tiger reserves.

Which of the above statements is/are correct?

 a) 1 only
 b) 2 only
 c) Both
 d) None

4.     Consider the following statements regarding State Disaster Response Fund (SDRF).

1.      SDRF was constituted based on the recommendations of the 14th Finance Commission.
2.      The Central Government contributes 50% to SDRF and the remaining 50% is provided from the State Government.
3.      A State Government may use the funds available under the SDRF for providing immediate relief to the victims of natural disasters that they consider to be ‘disasters’ within the local context in the State.

Which of the above statements is/are incorrect?

a)     1 only
b)     1, 2
c)      1, 3
d)     2, 3

5.     An estimated 4.2 million premature deaths globally are linked to ambient air pollution. Worldwide ambient air pollution can lead to

1.      Lung cancer
2.      Acute lower respiratory infection
3.      Stroke
4.      Ischaemic heart disease
5.      Chronic obstructive pulmonary disease

Select the correct answer code:

 a) 1, 2, 3 ,4
 b) 1, 3, 4, 5
 c) 2, 3, 4, 5
 d) 1, 2, 3, 4, 5

6.     Consider the following statements regarding Operation ‘Clean Art’.

1.      It is the first pan India operation to crackdown on the smuggling of mongoose hair.
2.      Operation Clean Art was conceived by Wildlife Crime Control Bureau.
3.      Any smuggling or possession of mongoose body part is a non-bailable offence.

Which of the above statements is/are correct?

a)     1, 2
b)     1, 3
c)      2, 3
d)     1, 2, 3

7.     Consider the following statements regarding Question Hour in the Indian Parliament.

1.      A Starred Question is one to which a member desires an oral answer from the Minister in the House and is required to be distinguished by him/her with an asterisk.
2.      An Unstarred Question is not called for oral answer in the House and no supplementary question can be asked thereon.
3.      As per Rules of Procedure and Conduct of Business in Lok Sabha, there are no Questions addressed to Private Members.

Which of the above statements is/are correct?

a)     1, 3
b)     2, 3
c)      1, 2
d)     1, 2, 3

8.     Consider the following statements regarding Common But Differentiated Responsibilities and Respective Capability (CBDR-RC).

1.      Common but Differentiated Responsibilities and Respective Capabilities (CBDR–RC) is a principle within the United Nations Framework Convention on Climate Change (UNFCCC).
2.      The UNFCCC divided countries into “Annex I” and “non-Annex I,”the former generally referring to developing countries and the latter to developed countries.
3.      CBDR-RC and the annex classifications were codified in the 1997 Kyoto Protocol.

Which of the above statements is/are correct?

a)     1, 2
b)     2, 3
c)      1, 3
d)     1 only

9.     Consider the following statements regarding Goldman Environmental Prize.

1.      The Prize recognizes individuals for sustained and significant efforts to protect and enhance the natural environment.
2.      The Prize was instituted by World Wide Fund for Nature(WWF).

Which of the above statements is/are correct?

a)     1 only
b)     2 only
c)      Both
d)     None

10.Who among the following are listed as the Particularly Vulnerable Tribal Groups in Andaman & Nicobar Islands.

1.      Jarawa
2.      Toda
3.      Sentinelese
4.      Shompen

Select the correct answer code:

e)     1, 2, 3
f)       2, 3, 4
g)     1, 3, 4
h)     1, 2, 3, 4

11.Consider the following statements regarding Instrument in Support of Trade Exchanges (INSTEX).

1.      The Instrument in Support of Trade Exchanges (INSTEX) is China’s payment mechanism established in 2019.
2.      Its mission is to facilitate non-USD transactions and non-SWIFT to avoid breaking U.S. sanctions.

Which of the above statements is/are correct?

a)     1 only
b)     2 only
c)      Both
d)     None

Image result for UPSC IMAGES
12.Consider the following statements regarding ‘Contract for the Web’.

1.      The ‘Contract for the Web’ is a United Nations document that aims to create a standard policy for a Web that benefits all.
2.      Governments who are looking to regulate in the digital era, can use the contract as a roadmap to lay out their policies and laws going forward.

Which of the above statements is/are correct?

a)     1 only
b)     2 only
c)      Both
d)     None

13. Emissions Gap Report measures

1.      The amount of greenhouse gas emissions every year up to 2030.
2.      The commitments countries are making to reduce their emissions and the impact these commitments are likely to have on overall emission reduction.
3.      The pace at which emissions must be reduced to reach an emission low that would limit temperature increase to 1.5 degree Celsius.

Which of the above statements is/are correct?

i)       1, 2
j)       1, 3
k)     2, 3
l)       1, 2, 3

14.Consider the following statements about Hangul, Kashmir’s iconic wildlife species.

1.      It is placed under Schedule I of the Indian Wildlife (Protection) Act, 1972.
2.      It is listed under the Species Recovery Programme of the Wildlife Institute of India (WII) and the Environmental Information System (ENVIS) of the MoEFCC.
3.      The state of Jammu & Kashmir, along with the IUCN and the WWF prepared a project for the protection of these animals, which was known as Project Hangul.

Which of the above statements is/are correct?

m)   1, 2
n)     2, 3
o)     1, 3
p)     1, 2, 3

15.Consider the following statements regarding Anaemia.

1.      Anaemia is a condition in which the number of red blood cells or their oxygen-carrying capacity is insufficient to meet physiologic needs.
2.      Protein deficiency is the most common cause of anaemia.
3.      Premature delivery can also lead to childhood anaemia.

Which of the above statements is/are correct?

a)     1, 2
b)     2, 3
c)      1, 3
d)     1 only

16.Consider the following statements regarding Alliance of Small Island States (AOSIS).

1.      Alliance of Small Island States (AOSIS) is an intergovernmental organization of low- lyingcoastal and small island countries.
2.      The main purpose of the alliance is to consolidate the voices of Small Island Developing States (SIDS) to address global warming.
3.      AOSIS partners with United Nations Development Programme (UNDP) to effectively influence climate negotiations.

Which of the above statements is/are correct?

a)     1, 2
b)     1, 3
c)      2, 3
d)     1, 2, 3

17.Which of the following statement is correct regarding Sustainable Alternative Towards Affordable Transportation (SATAT) initiative.

a)     Sub component under FAME India Phase II for promotion of Electric Mobility in the country.
b)     Dedicated lanes for buses in urban areas for improving the reliability of public transport services.
c)      Setting up of Compressed Bio-Gas production plants and make it available for use in automotive fuels.
d)     None of the above

18.The first multilateral bloc to declare climate emergency is

a)     ASEAN
b)     BIMSTEC
c)      European Union (EU)
d)     G20 Members

19.Consider the following statements about Asia-Pacific awards for cultural heritage conservation.

1.      It is awarded by UNESCO.
2.      It recognises the efforts taken to restore and conserve historical structures without affecting their heritage value.
3.      New Delhi has won most recognitions in India since the inception of the awards.

Which of the above statements is/are correct?

a)     1, 3
b)     2, 3
c)      1, 2
d)     1, 2, 3

20.Consider the following statements regarding Carbon offsetting.

1.      Carbon offsetting allows a country to help reach its own emissions reduction targets by funding emission reductions in another country.
2.      The United Nation’s Clean Development Mechanism (CDM) set up under the 1997 Kyoto Protocol is first major Carbon offsetting scheme.
3.      Carbon offsets can be bought by individuals, companies or countries.

Which of the above statements is/are correct?

a)     1, 2
b)     1, 3
c)      2, 3
d)     1, 2, 3

21.Consider the following statements regarding “Power of Siberia” gas pipeline.

1.      Power of Siberia is the first cross-border gas pipeline between Russia and China.
2.      The pipeline passes through China’s Yangtze River.

Which of the above statements is/are correct?

a)     1 only
b)     2 only
c)      Both
d)     None


22.Consider the following statements regarding YuWaah initiative.

1.      It is launched by UNESCO.
2.      It is a multi-stakeholder alliance which aims to facilitate youth inside and outside formal education systems to gain relevant skills for productive lives and the future of work.
3.      It brings young people together with the private sector, governments, international and local organisations.

Which of the above statements is/are correct?

a)     1, 2
b)     1, 3
c)      2, 3
d)     1, 2, 3

23.Consider the following statements regarding Global Migration Report 2020.

1.      ‘Global Migration Report 2020’ has been released by International Organisation for Migration (IOM).
2.      After China, India accounts for the next highest share of its diaspora living outside the country.
3.      Remittances received by India have consistently increased since 2005.

Which of the above statements is/are correct?

a)     1, 2
b)     1, 3
c)      2, 3
d)     1, 2, 3

24.Consider the following statements about Treaty on the Prohibition of Nuclear Weapons (TPNW).

1.      It is the first legally binding international agreement to comprehensively prohibit nuclear weapons.
2.      It was passed in 1974 after India conducted its first nuclear test.
3.      The treaty prohibits the development, testing, production, stockpiling, transfer and use of nuclear weapons for all the UN member countries.

Which of the above statements is/are correct?

a)     1 only
b)     1, 2
c)      1, 3
d)     1, 2, 3

25.Consider the following statements regarding Bharat Bond Exchange Traded Fund (ETF).

1.      Bharat Bond Exchange Traded Fund (ETF), is the country’s first Government bond exchange-traded fund aimed at retail investors.
2.      Individuals, non-resident Indians, high networth individuals, top tier corporates and foreign portfolio investors (FPIs) can invest in Bharat Bond Exchange Traded Fund (ETF).

Which of the above statements is/are correct?

(a)  1 only
(b)  2 only
(c)   Both
(d)  None

26.Consider the following statements with respect to ‘National Investigation Agency’

1.      It is under the jurisdiction of the Ministry of Personnel, Public Grievances and Pensions.
2.      It is established under The Delhi Special Police Establishment (DSPE) Act, 1946.

Select the correct statements

a)     1 Only
b)     2 Only
c)      Both
d)     None


27.Consider the following statements with respect to ‘Indian Skimmer’.

1.      It is listed as ‘Vulnerable’ under the IUCN Red List.
2.      It is found in the Indian Peninsular Region.

Select the correct statements

a)     1 Only
b)     2 Only
c)      Both
d)     None
 Image result for UPSC IMAGES
28.Consider the following statements with respect to ‘National Pharmaceutical Pricing Authority (NPPA)’

1.      It is under the aegis of the Ministry of Health and Family Welfare.
2.      It implements and enforces the provisions of the Drugs (Prices Control) Order in accordance with the powers delegated to it.

Select the correct statements

a)     1 Only
b)     2 Only
c)      Both
d)     None

29.Consider the following statements with respect to ‘2I/Borisov’

1.      It is the first observed interstellar comet.
2.      It was observed by the Pan-STARRS telescope.

Select the correct statements

 a) 1 Only
 b) 2 Only
 c) Both
 d) None

30.NASA’s ‘OSIRIS-Rex’ Mission is associated with

a)     Oumuamua
b)     Bennu
c)      Pluto
d)     Mars

31.Consider the following statements with respect to ‘Climate Ambition Alliance’

1.      It will focus on the submission of enhanced Nationally Determined Contributions; reaching new commitments to achieve Net Zero by 2050; and the implementation of measures to strengthen the protection of forests and oceans.
2.      It was launched at the Climate Action Summit in New York and is led by Chile.

Select the correct statements

a)     1 Only
b)     2 Only
c)      Both
d)     None

32.‘JAGA’ Mission was in news recently. It is associated with which of the following states?

a)     Andhra Pradesh
b)     Rajasthan
c)      Maharashtra
d)     Odisha

33. ‘East Mediterranean Gas Forum’ was not signed by

a)     Israel
b)     Turkey
c)      Egypt
d)     Jordan

34.‘Thomas Committee’ is associated with

a)     Prevention of Damage to Public Property
b)     Foreign Exchange Management and Regulation
c)      Prevention of Money Laundering
d)     Prevention of Terrorist and Disruptive Activities

35.‘The Global Gender Gap Report’ is published by

a)     World Economic Forum
b)     International Labour Organisation
c)      UN Women
d)     UNDP

36.The ‘Hong Kong Convention’ is associated with

a)     Ship Recycling
b)     Rights of Persons with Disabilities
c)      Laws of war and war crimes
d)     Status of Refugees

37.Consider the following statements with respect to ‘BrahMos’.

1.      It is a joint venture of the DRDO of India and the NPOM of Russia.
2.      It is regarded as the fastest supersonic cruise missile in the world and is operational with the Indian Army, Navy and Air Force.

Select the correct statements

a)     1 Only
b)     2 Only
c)      Both
d)     None

38.Consider the following statements with respect to ‘Global Refugee Forum’

1.      It is guided by the Global Compact on Refugees and is held in Geneva, Switzerland.
2.      The inaugural ‘GRF’ was held in 2016 at the backdrop of the Syrian Refugee Crisis.

Select the correct statements

a)     1 Only
b)     2 Only
c)      Both
d)     None

39.Consider the following statements with respect to ‘African swine fever (ASF)’

1.      It is a disease listed in the World Organisation for Animal Health (OIE) Terrestrial Animal Health Code and must be reported to the OIE.
2.      ASF is not a risk to human health.

Select the correct statements

a)     1 Only
b)     2 Only
c)      Both
d)     None

40.‘Himalayan gold’ was in news recently. It is referred to

a)     Caterpillar Fungus
b)     Sal Tree
c)      Rhododendron
d)     Cannabis ruderalis

41.Consider the following statements with respect to ‘Pradhan Mantri Gram Sadak Yojana’

1.      Following the recommendations of the 14th Finance Commission, the scheme is funded by both the central government (60%) and states (40%).
2.      It is under the aegis of the Ministry of Road Transport and Highways.

Select the correct statements

a)     1 Only
b)     2 Only
c)      Both
d)     None

42.‘Santamasa’, was in news recently. What is it?

a)     Exoplanet
b)     Constellation
c)      Comet
d)     Asteroid

43. The ‘Silver Line Railway Project’ is associated with which of the following states?

a)     Maharashtra
b)     Kerala
c)      Arunachal Pradesh
d)     Assam

44.Highest number of refugees come from which of the following countries?

a)     South Sudan
b)     Syria
c)      Myanmar
d)     Afghanistan


45. ‘Trastuzumab’ is biosimilar launched by WHO to treat

a)     HIV/AIDS
b)     Breast Cancer
c)      Rotavirus
d)     Japanese encephalitis


46.Consider the following statements with respect to ‘Pinaka Missile’.

1.      The system has a maximum range of 40 km for Mark-I and 75 km for Mark-II.
2.      It is developed by the Reliance Naval and Engineering Limited (R-Naval).

Select the correct statements

a)     1 Only
b)     2 Only
c)      Both
d)     None

47.‘Pollution and Health Metrics: Global, Regional and Country Analysis’ 2019 report is published by

a)     World Health Organisation (WHO)
b)     Global Alliance on Health and Pollution (GAHP)
c)      UN Environment Programme (UNEP)
d)     Intergovernmental Panel on Climate Change

48.Which of the following are transboundary rivers between India and Bangladesh?

1.      Teesta
2.      Ganga
3.      Brahmaputra

Select the correct code:

a)     1 and 2
b)     2 and 3
c)      1 and 3
d)     All of the above

49.Consider the following statements with respect to ‘Exercise ‘Apharan’

1.      It is a large-scale humanitarian assistance and disaster relief (HADR) exercise.
2.      It is conducted by the Indian Navy in collaboration with Indian Coast Guard.

Select the correct statements

a)     1 Only
b)     2 Only
c)      Both
d)     None
 Image result for UPSC IMAGES
50.Consider the following statements with respect to ‘MacQueen’s bustard’

1.      It is also known as the ‘African Houbara’.
2.      It is listed as ‘Critically Endangered’ under the IUCN Red List.

Select the correct statements

 a) 1 Only
 b) 2 Only
 c) Both
 d) None

51.Which of the following statements are correct?

1.      The Central Industrial Security Force (CISF) is a Central Armed Police Force in India.
2.      CISF is directly under the Union Ministry of Defence.

Which of the following statement/s is/are correct?

a. 1 only
b. 2 only
c. Both 1 and 2
d. Neither 1 nor 2

52.Which of the following statements are correct?

1.      The INS Arihant and INS Varsha are the two nuclear-powered submarines currently in service with the Navy.
2.      Sagarika is a nuclear-capable submarine-launched ballistic missile.

a)     1 only
b)     2 only
c)      Both 1 and 2
d)     Neither 1 nor 2

53.Which of the following statements are correct?

1.      The Juvenile Justice (Care and Protection of Children) Act, 2015 provides legal immunity to juvelines under 18 years of age in all cases.
2.      The Juvenile Justice (Care and Protection of Children) Act, 2015 does not deal with adoption and associated issues.

a. 1 only
b. 2 only
c. Both 1 and 2
d. Neither 1 nor 2

54.Which of the following constitute the Foreign Reserves of India?

1.      Foreign Currency Assets Gold held with Bank of International settlements
2.      Reserve Tranche Position with World Bank Special Drawing Rights (SDRs) with International Monetary Fund

Which of the following statement/s is/are correct?

a. 1 only
b. 2 only
c. Both 1 and 2
d. Neither 1 nor 2


55.Consider the following statements with respect to National Population Register (NPR):

1.      NPR is a register of permanent residents of the country.
2.      Under the provisions of NPR, a resident identity card (RIC) will be issued to individuals only over the age of 18.

Which of the following statement/s is/are correct?

a)     1 only
b)     2 only
c)      Both 1 and 2
d)     Neither 1 nor 2

56. “Operation Twist” refers to:

a)     The name given to the monetary policy operation that involves the purchase and sale of bonds.
b)     The operation by the Military of India that led to the capture of Goa, Daman and Diu and Anjediva Islands.
c)      The operation of the Indian Armed Forces to capture the Siachen Glacier in the Kashmir region.
d)     None of the above.



57. Consider the following statements with respect to Communications Compatibility and Security Agreement (COMCASA):

1.      It provides a legal framework for the transfer of communication security equipment from the US to India.
2.      It is one of the four foundational agreements that a country needs to sign to become a major defence partner of the United States.

Which of the given statement/s is/are correct?

a)     1 only
b)     2 only
c)      Both 1 and 2
d)     Neither 1 nor 2

58. Consider the following statements with respect to INDRA 2019:

1.      INDRA 2019 is a joint tri-services exercise between India and Russia.
2.      When the series of exercise began, it was conducted as a single service exercise alternately between the two countries.

Which of the given statement/s is/are correct?

a)     1 only
b)     2 only
c)      Both 1 and 2
d)     Neither 1 nor 2

59. Consider the following statements:

1.      There is no time limit for the President to respond to a mercy petition.
2.      The mercy petition is not subject to Judicial Review.
3.      In India, the authority to decide on mercy petitions rests with the President and the Governor.

Which among the above statements is/are correct?

a)     1 and 2 only
b)     1 and 3 only
c)      2 and 3 only
d)     1, 2 and 3

60. Consider the following statements:

1.      Bangladesh shares 57 trans-boundary rivers with India.
2.      Bangladesh is surrounded on three sides by India.
3.      The country has a gigantic delta formed by the alluvial deposits of the Ganges, the Brahmaputra and the Meghna.

Which of the given statement/s is/are correct?

a)     1 and 2 only
b)     2 and 3 only
c)      2 only
d)     1 and 3 only

61. Consider the following:

1.      Remission: Reduction of punishment without changing the nature of the punishment.
2.      Pardon: Completely absolving the person of the crime and letting him/her go free.
3.      Commute: Reduction of the type of punishment into a less harsh one.

Which of the above is/are correct?

a)     1 only
b)     1 and 3 only
c)      2 and 3 only
d)     1, 2 and 3

62. Consider the following statements with respect to the National Company Law Appellate Tribunal (NCLAT):

1.      It hears appeals against the orders of the National Company Law Tribunal(s) (NCLT).
2.      It is the Appellate Tribunal for hearing appeals against the orders passed by the Insolvency and Bankruptcy Board of India.
3.      It is the Appellate Tribunal to hear and dispose of appeals against any direction issued or decision made or order passed by the Competition Commission of India (CCI).

Which of the given statement/s is/are incorrect?

a)     1 only
b)     3 only
c)      1 and 2 only
d)     None of the above


63. Consider the following statements:
1.      Solar Eclipse can only occur on a Full Moon day.
2.      In case of a Solar Eclipse, the Moon passes between Earth and Sun and the Moon’s shadow happens to fall upon Earth’s surface.

Which of the given statement/s is/are correct?

a)     1 only
b)     2 only
c)      Both 1and 2
d)     Neither 1 nor 2


64. “Red Octagon” recently seen in news is:
A warning symbol employed in packaged foods.

a)     A major area in the basin of the Pacific Ocean where many earthquakes and volcanic eruptions occur.
b)     An autonomous territory in northwest
c)      China that is home to Turkic Uyghur people.
d)     None of the above.

65. Consider the following statements with respect to Maternal Mortality:

1.      Maternal mortality refers to deaths due to complications from pregnancy or childbirth.
2.      Maternal mortality ratio is calculated per 10,000 live births.
3.      India has seen a declining trend of Maternal Mortality Rate since the year 2007.

Which of the given statement/s is/are incorrect?

a)     3 only
b)     2 only
c)      1 and 2 only
d)     2 and 3 only

66. Consider the following statements with respect to Missile Technology Control Regime (MTCR):

1.      It is a multilateral export control regime.
2.      It seeks to encourage the proliferation of missiles and missile technology.
3.      India is a member of MTCR.

Which of the given statement/s is/are correct?

a)     1 only
b)     1 and 2 only
c)      1 and 3 only
d)     1, 2 and 3

67. Consider the following statements with respect to the Citizenship Amendment Act:

1.      The Act gives eligibility for Indian citizenship to illegal migrants who are Hindus, Sikhs, Buddhists, Jains, Parsis and Christians from Afghanistan, Myanmar and Pakistan.
2.      The Act does not apply to the tribal areas of Assam, Meghalaya, Tripura, Arunachal Pradesh, Mizoram and Nagaland.

Which of the given statement/s is/are correct?

a)     1 only
b)     2 only
c)      Both 1 and 2
d)     Neither 1 nor 2


68. Consider the following statements with respect to Light Detection and Ranging (LiDAR):

1.      LiDAR is a remote sensing method that uses light in the form of a pulsed laser to measure variable distances to the Earth.
2.      Topographic LiDAR typically uses a near-infrared laser to map the land, while bathymetric LiDAR uses water-penetrating green light to also measure seafloor and riverbed elevation.
3.      LiDAR can be used for monitoring pollution.

Which of the given statement/s is/are correct?

a)     1 and 2 only
b)     2 and 3 only
c)      1, 2 and 3
d)     1 and 3 only

69. Consider the following statements with respect to NIRVIK:

1.      The aim of the scheme is to ease the lending process and enhance loan availability for exporters.
2.      The scheme was introduced by the Export Credit Guarantee Corporation of India (ECGC).
3.      The insurance cover guaranteed under the scheme will cover up to 90 percent of the principal and interest.

Which of the given statement/s is/are incorrect?

a)     1 and 2 only
b)     1, 2 and 3
c)      2 and 3 only
d)     None of the above

70. “SURYA KIRAN” is a joint military training exercise between:

a)     India – China
b)     India – Nepal
c)      India – Sri Lanka
d)     India – Bangladesh


71. Which of the following pairs of martial arts and associated states is wrongly matched?

a)     Mukna: Bihar
b)     Huyen Langlon: Manipur
c)      Silambam: Tamil Nadu
d)     Paika Akhada: Odisha

72. Which of the statements is/are correct?

1.      Citizenship is listed under the union list of the 7th Schedule of the Indian Constitution.
2.      Article 256 of the Constitution states that the executive power of every State shall be so exercised as to ensure compliance with the laws made by Parliament.

Options:

a)     1 only
b)     2 only
c)      Both 1 and 2
d)     Neither 1 nor 2

73. Which of the following is wrongly matched?

a)     Araku Valley Arabica coffee: Andhra Pradesh
b)     Kani shawl: Jammu and Kashmir
c)      Muga silk: Assam
d)     Toda embroidery: Madhya Pradesh

74. Which of the following statements is/are correct?

1.      The Maternity Benefit Amendment Act is applicable only for the biological mothers and not those who are adopting a child.
2.      The Maternity Benefit Amendment Act makes crèche facility mandatory for every establishment employing 10 or more employees.

Options:

a)     1 only
b)     2 only
c)      Both 1 and 2
d)     Neither 1 nor 2

75.With reference to the Global Sulphur cap complaint fuel oil, consider the following statements:

1.      International Maritime Organization (IMO) regulations to reduce sulphur oxides (SOx) emissions from ships first came into force in 2005, under Annex VI of the International Convention for the Prevention of Pollution from Ships (known as the MARPOL Convention).
2.      From 1 January 2020, the limit for sulphur in fuel oil used on board ships operating outside designated emission control areas will be reduced to 0.50% m/m (mass by mass).

Which of the statements given above is/are correct?

a)                 1 only
b)                 2 only
c)                  Both 1 and 2
d)                 Neither 1 nor 2


76.With reference to the Article 30(1) of the Constitution of India, consider the following statements:

1.      Article 30(1) of the Constitution of India gives linguistic and religious minorities a fundamental right to establish and administer educational institutions of their choice.
2.      The Supreme Court in T.M.A. Pai Foundation vs. State of Karnataka (2002) held that a minority, whether linguistic or religious, is determinable only by taking into consideration the population of the country as a whole.

Which of the statements given above is/are correct?

a)                 1 only
b)                 2 only
c)                  Both 1 and 2
d)                 Neither 1 nor 2

77.With reference to the ‘COP25’ 2019 United Nations Climate Change Conference, consider the following statements:

1.      International climate talks at COP25 closed with no deal on carbon markets.
2.      It was held in Chile under the presidency of the Chilean government.

Which of the statements given above is/are correct?

a)                 1 only
b)                 2 only
c)                  Both 1 and 2
d)                 Neither 1 nor 2

78.With reference to the Sustainable Development Cell (SDC), consider the following statements:

1.      Union Ministry of Coal has recently decided to establish a Sustainable Development Cell (SDC) to promote environmentally sustainable coal mining in the country.
2.      It will address environmental concerns during the decommissioning or closure of mines.

Which of the statements given above is/are correct?

a)                    1 only
b)                    2 only
c)                     Both 1 and 2
d)                    Neither 1 nor 2

79.Consider the following statements:

1.      Central Registration Centre for name reservation and incorporation of companies is an initiative of Union Ministry of Finance.
2.      Provisions relating to creation of Debenture Redemption Reserve (DRR) have been revised by the Ministry of Corporate Affairs with the objective of deepening the bond market & reducing the cost of capital.

Which of the statements given above is/are correct?

a)                 1 only
b)                 2 only
c)                  Both 1 and 2
d)                 Neither 1 nor 2


80.What is ‘StrandHogg’, recently seen in news?

a)                 Bug in android applications
b)                 Newly found exoplanet
c)                  Crypto currency of Peru
d)                 None of the above

81..With reference to the hydrogen fuel cell, consider the following statements:

1.      The fuel cell combines hydrogen and oxygen to generate an electric current, water being the only byproduct.
2.      Fuel cells produce much smaller quantities of greenhouse gases and none of the air pollutants that cause health problems.

Which of the statements given above is/are correct?

A.         1 only
B.         2 only
C.         Both 1 and 2
D.         Neither 1 nor 2

82.With reference to the Government Instant Messaging System (GIMS), consider the following statements:

1.      It is being packaged for employees of Central and state government departments and organisations for intra and inter organisation communications.
2.      It is designed and developed by the DRDO.

Which of the statements given above is/are correct?

a)                 1 only
b)                 2 only
c)                  Both 1 and 2
d)                 Neither 1 nor 2

83. The Māori are the indigenous Polynesian people of which of the following country?

a)                 New Zealand
b)                 France
c)                  Spain
d)                 Italy

84. With reference to the Integral Coach Factory (ICF), consider the following statements:
1.      Integral Coach Factory (ICF) is a manufacturer of rail coaches located in the neighbourhood of Perambur, Chennai, Tamil Nadu.
2.      It was established in 1940.

Which of the statements given above is/are correct?

a)           1 only
b)           2 only
c)            Both 1 and 2
d)           Neither 1 nor 2

85.Consider the following statements with respect to Quality Council of India (QCI)

1.      The Department for Promotion of Industry and Internal Trade, Ministry of Commerce & Industry, is the nodal ministry for QCI.
2.      It was set up jointly by the Government of India and the Indian Industry represented by the premier industry associations.
3.      Chairman of QCI is appointed by the Prime Minister.

Which of the statement(s) given above is/are correct?

a)     1 and 2 only
b)     2 and 3 only
c)      1 and 3 only
d)     1,2 and 3 only

86.Consider the following statements with respect to International Human Solidarity Day

1.      Solidarity is identified in the Millennium Declaration as one of the fundamental values of International relations.
2.      The UN General Assembly, by Resolution 377A, proclaimed  International Human Solidarity Day.

Which of the statement(s) given above is/are correct?

a)     1 only
b)     2 only
c)      Both 1 and 2
d)     Neither 1 nor 2

87.Consider the following statements with respect to International Astronomical Union (IAU)

1.      It was founded in 1919 and serves as the internationally recognized authority for assigning designations to celestial bodies and surface features on them.
2.      IAU recentlynamed a new star 'Sharjah' and its planet ‘Barajeel'.

Which of the statement(s) given above is/are not correct?

a)     1 only
b)     2 only
c)      Both 1 and 2
d)     Neither 1 nor 2

88.India’s first ‘Waste Exchange Platform’ was launched in,

a)     Kochi
b)     Indore
c)      Mysuru
d)     Chennai

89.Consider the following statements with respect to European Green Deal

1.      The overarching objective of the European Green Deal aims to reach net-zero greenhouse gas emissions by 2050.
2.      It includes a chemical strategy for a toxic-free environment.
3.      Building renovation is meant to be one of the flagship programmes of the Green Deal.

Which of the statement(s) given above is/are correct?

a)     1 and 2 only
b)     1 and 3 only
c)      2 and 3 only
d)     1,2 and 3 only

90.Consider the following statements with respect to EChO Network

1.      It aims to provide a template for cross-disciplinary leadership in India.
2.      It was launched by Microsoft in association with Ministry of Home Affairs.

Which of the statement(s) given above is/are correct?

a)     1 only
b)     2 only
c)      Both 1 and 2
d)     Neither 1 nor 2

91.“Green Channel Clearance” sometimes seen in the news recently is associated with which of the following agency?

a)     Airports Authority of India
b)     Competition Commission of India
c)      Securities and Exchange Board of India
d)     Genetic Engineering Appraisal Committee

92.Consider the following statements with respect to Indian Pharmacopoeia (IP)

1.      It is an officially recognized book of standards as per the Drugs Price Control Order 1995 and rules 2005 thereunder.
2.      Afghanistan is the first country to recognize Indian Pharmacopoeia.

Which of the statement(s) given above is/are correct?

a)     1 only
b)     2 only
c)      Both 1 and 2
d)     Neither 1 nor 2

93.“Mole lander” sometimes seen in the news recently is associated with which of the following missions?

a)     NASA’s InSight Mission
b)     ISRO’s Mangalyaan 2
c)      NASA’s Juno Mission
d)     ESA’s Rosetta Mission

94.With respect to Macau, consider the following statements

1.      Macau is a special administrative region of China, on the country’s western coast.
2.      It uses the same political model as Hong Kong - "One country, Two systems".

Which of the statement(s) given above is/are correct?

a)     1 only
b)     2 only
c)      Both 1 and 2
d)     Neither 1 nor 2


95.Consider the following statements with respect to 2+2 Ministerial Dialogue 2019

1.      It is the highest-level institutional mechanism between U.S.A and India held annually.
2.      It was the second 2+2 meeting and the first to be held in New Delhi.

Which of the statement(s) given above is/are correct?

(a)  1 only
(b)  2 only
(c)   Both 1 and 2
(d)  Neither 1 nor 2

96.Consider the following statements with respect to Pinaka Missile System

1.      It is an Artillery Missile System capable of striking into enemy territory up to a range of 250 kilometres with high precision.
2.      It was jointly developed by Defence Research and Development Organisation (DRDO) and Russian Defence Research Agency (RDRA).
3.      The navigation system of the missile is aided by the GPS Aided GEO Augmented Navigation (GAGAN), which is also used in civil aviation applications in India.

Which of the statement(s) given above is/are incorrect?

a)     1 only
b)     2 only
c)      1, 2 and 3
d)     None

97.Consider the following statements with respect to Investigation Tracking System for Sexual Offences (ITSSO)

1.      It is an online module available to law enforcement agencies at all levels- National, State, District and Police Station.
2.      It allows State to undertake real-time monitoring and management for completion of investigation in rape cases in 2 months.
3.      It was launched by the Ministry of Home Affairs and it leverages the existing CCTNS data base.

Which of the statement(s) given above is/are correct?

a)     1 only
b)     1 and 3 only
c)      1, 2 and 3
d)     None

98.“Operation Twist” sometimes seen in the news recently is associated with which of the following?

a)     Reserve Bank of India
b)     Indian Coast Guard
c)      Defence Research and Development Organisation
d)     Ministry of Home Affairs

99."Jalsathi programme" sometimes seen in the news recently was launched recently by which of the following states?

a)     Odisha
b)     Gujarat
c)      Karnataka
d)     Maharashtra

100.       Consider the following statements with respect to Section 144 of the Code Of Criminal Procedure (CrPC), 1973

1.      It can be used to restrict even a single individual.
2.      It authorises only the district magistrate  to issue orders to prevent and address urgent cases of apprehended danger or nuisance.

Which of the statement(s) given above is/are correct?

a)     1 only
b)     2 only
c)      Both 1 and 2
d)     Neither 1 nor 2

101.       Consider the following statements with respect to Directorate General of Foreign Trade (DGFT)

1.      It is an attached office of the Ministry of Finance.
2.      It has been assigned the role of ‘facilitator’, to promote and facilitate exports/imports, of the country.

Which of the statement(s) given above is/are correct?

a)     1 only
b)     2 only
c)      Both 1 and 2
d)     Neither 1 nor 2

102.       Consider the following statements with respect to Gay conversion therapy

1.      It purports to cure homosexuality by turning gay people, straight.
2.      It uses psychotherapeutic means or counselling to eliminate an individual’s desire for members of their own sex.
3.      It sees homosexuality as a mental disorder.

Which of the statements given above are correct?

a)     1 and 2 only
b)     2 and 3 only
c)      1 and 3 only
d)     1, 2 and 3

103.        ‘Kambala’ an annual traditional sport is being held in?

a)     Kerala
b)     Telangana
c)      Karnataka
d)     Odisha

104.       Uzbekistan  shares its land border with which of the following countries

1.      Tajikistan,
2.      Afghanistan,
3.      Turkmenistan,
4.      Kazakhstan,
5.      Kyrgyzstan

Choose the correct option

a)     1, 2, 3 and 5 only
b)     1, 4 only 5 only
c)      1, 3, 4 and 5 only
d)     1, 2, 3, 4 and 5


 ANSWER KEY 
WITH EXPLACATION


1.     Solution: d)
The Indian Army has inducted Israel-made anti-tank guided missiles (ATGMS) Spike along the LoC in northern command theatre in Jammu and Kashmir to bolster defence along the country’s border with Pakistan.
Known as the “fire and forget” missiles, Spike ATGMs are portable by men and are powerful enough to destroy tanks and bust bunkers within four kilometers.
The “fire and forget” ATGMs have a strike range of up to four kilometers and can be used to destroy bunkers, shelters, launching pads and terrorist training camps close to the LoC.
The missiles can be deployed in both mountains and plains, they can be fired from different platforms, including vehicles, helicopters, ships and ground launchers.
2.     Solution: b)
Cartosat-3 satellite is a third-generation agile advanced satellite having high resolution imaging capability.
Cartosat-3 could be potentially used for weather mapping and cartography. It aims to address the increased demands for large scale urban planning, rural resource and infrastructure development, coastal land use and land cover.
3.     Solution: b)
Guru Ghasidas National Park:
This will be Chattisgarh’s 4th tiger reserve.
Current 3 tiger reserves: Achanakmar, Udanti- Sitanadi and Indravati tiger reserves. The National Tiger Conservation Authority (NTCA) had approved to declare Guru Ghasidas National Park as a tiger reserve.
4.     Solution: b)
SDRF has been constituted by each state under the provisions of Disaster Management act 2005.
It was constituted based on the recommendations of the 13th Finance Commission.
Funding: The government of India contributes 75% and 90% of the total yearly allocation of SDRF to general states and special category states respectively.
Heads: The state executive committee headed by the Chief Secretary is authorized to decide on all matters relating to the financing of the relief expenditure from the SDRF.
Disaster (s) covered under SDRF: Cyclone, drought, earthquake, fire, flood, tsunami, hailstorm, landslide, avalanche, cloudburst, pest attack, frost and cold waves.
Local Disaster: A State Government may use up to 10 percent of the funds available under the SDRF for providing immediate relief to the victims of natural disasters that they consider to be ‘disasters’ within the local context in the State and which are not included in the notified list of disasters of the Ministry of Home Affairs subject to the condition that the State Government has listed the State specific natural disasters and notified clear and transparent norms and guidelines for such disasters with the approval of the State Authority, i.e., the State Executive Authority (SEC).
5.     Solution: d)
An estimated 4.2 million premature deaths globally are linked to ambient air pollution, mainly from heart disease, stroke, chronic obstructive pulmonary disease, lung cancer, and acute respiratory infections in children.
Worldwide ambient air pollution accounts for:
29% of all deaths and disease from lung cancer
17% of all deaths and disease from acute lower respiratory infection
24% of all deaths from stroke
25% of all deaths and disease from ischaemic heart disease
43% of all deaths and disease from chronic obstructive pulmonary disease
6.     Solution: d)
Operation ‘Clean Art’:
It is first pan India operation to crackdown on the smuggling of mongoose hair.
Conceived by WCCB.
Aim: To ensure that the mongoose hair brush trade should be closed down across the country.
Protection: Mongoose is listed in Schedule II Part 2 of the Wildlife Protection Act. Any smuggling or possession of its body part is a non-bailable offence.
7.     Solution: c)
Members have a right to ask questions to elicit information on matters of public importance within the special cognizance of the Ministers concerned. The questions are of four types:
Starred Questions: A Starred Question is one to which a member desires an oral answer from the Minister in the House and is required to be distinguished by him/her with an asterisk. Answer to such a question may be followed by supplementary questions by members.
Unstarred Questions: An Unstarred Question is one to which written answer is desired by the member and is deemed to be laid on the Table of the House by Minister. Thus it is not called for oral answer in the House and no supplementary question can be asked thereon.
Short Notice Questions: A member may give a notice of question on a matter of public importance and of urgent character for oral answer at a notice less than 10 days prescribed as the minimum period of notice for asking a question in ordinary course. Such a question is known as ‘Short Notice Question’.
Questions to Private Members: A Question may also be addressed to a Private Member (Under Rule 40 of the Rules of Procedure and Conduct of Business in Lok Sabha), provided that the subject matter of the question relates to some Bill, Resolution or other matter connected with the business of the House for which that Member is responsible. The procedure in regard to such questions is same as that followed in the case of questions addressed to a Minister with such variations as the Speaker may consider necessary.

8.     Solution: c)
Common but Differentiated Responsibilities and Respective Capabilities (CBDR–RC) is a principle within the United Nations Framework Convention on Climate Change (UNFCCC).
It acknowledges the different capabilities and differing responsibilities of individual countries in addressing climate change.
Reflecting CBDR-RC, the Convention divided countries into “Annex I” and “non-Annex I,” the former generally referring to developed countries and the latter to developing countries.
Under the Convention Annex I countries have a greater mitigation role than non Annex-I countries.
CBDR-RC and the annex classifications were codified in the 1997 Kyoto Protocol, and Annex I country emissions reductions were legally bound.

9.     Solution: a)
The Goldman Environmental Prize honors grassroots environmental heroes from the world’s six inhabited continental regions: Africa, Asia, Europe, Islands & Island Nations, North America, and South & Central America.
The Prize recognizes individuals for sustained and significant efforts to protect and enhance the natural environment, often at great personal risk.
The Goldman Prize views “grassroots” leaders as those involved in local efforts, where positive change is created through community or citizen participation in the issues that affect them. Through recognizing these individual leaders, the Prize seeks to inspire other ordinary people to take extraordinary actions to protect the natural world. 2019 marks the 30th anniversary of the Prize founded in 1989 by U.S. philanthropists Rhoda and Richard Goldman.
10.Solution: c)
Name of the Particularly Vulnerable Tribal Group(s) in Andaman & Nicobar Islands – Great Andamanese, Jarawa, Onge, Sentinelese and Shompen.
Toda in Tamil Nadu

11.Solution: b)
Six Countries – Belgium, Denmark, Finland, the Netherlands, Norway and Sweden have recently joined INSTEX.
What is it?
It is a payment mechanism being setup by the European Union to secure trade with Iran and skirt US sanctions after Washington pulled out of the landmark nuclear deal last May.
It is registered at Paris with an initial 3,000 Euros in the capital and a supervisory board with members from France and Germany and chaired by the UK.
Key features of INSTEX:
It will allow trade between the EU and Iran without relying on direct financial transactions.
It will initially be used for non-sanctionable trade, including humanitarian goods such as medicine, food and medical devices.

12.Solution: b)
Sir Tim Berners-Lee, inventor of the World Wide Web, has announced a “Contract for the Web” — aimed at saving the future of his invention.
The idea is to create a global plan of action for all stakeholders to together commit to building a “better” Web. The goal is to create a standard policy for a Web that benefits all.
The Contract consists of nine principles — three each for governments, private companies, and individuals and civil society to endorse.
It has been created by representatives from over 80 organisations, including governments, companies, civil society activists, and academics.
The Contract is not meant to be “simply aspirational”, or just a “declaration”. “It’s actually meant to be implemented, and it’s meant to be a plan of action.
Governments who are looking to regulate in the digital era, can use the contract as a roadmap to lay out their policies and laws going forward.

13.Solution: d)
This Emissions Gap Report from UNEP examines the progress of countries to close the gap via their commitments to emissions reduction, to ultimately stop climate change.
The Emissions Gap Report measures and projects three key trendlines:
The amount of greenhouse gas emissions every year up to 2030
The commitments countries are making to reduce their emissions and the impact these commitments are likely to have on overall emission reduction
The pace at which emissions must be reduced to reach an emission low that would limit temperature increase to 1.5oC, affordably
The report also identifies key opportunities for each country to increase the pace of emission reduction necessary to close the gap.

14.Solution: d)
A massive decline in the population of Kashmir’s iconic wildlife species, the Hangul (Cervus hanglu hanglu), also known as the Kashmir stag, continues to be a big concern.
It is the state animal of Jammu & Kashmir.
It is found in dense riverine forests in the high valleys and mountains of the Kashmir Valley and northern Chamba district in Himachal Pradesh. In Kashmir, it’s found in the Dachigam National Park
It is placed under Schedule I of the IndianWildlife (Protection) Act, 1972.
The Hangul was once widely distributed inthe mountains of Kashmir and parts of Chamba district in neighbouring Himachal Pradesh.
The IUCN’s Red List has classified it as Critically Endangered and is similarly listed under the Species Recovery Programme of the Wildlife Institute of India (WII) and the Environmental Information System (ENVIS) of the MoEFCC.
The state of Jammu & Kashmir, along with the IUCN and the WWF prepared a project for the protection of these animals, which was known as Project Hangul.

15.Solution: c)
Anaemia is a condition in which the number of red blood cells or their oxygen-carrying capacity is insufficient to meet physiologic needs, which vary by age, sex, altitude, smoking, and pregnancy status.
Iron deficiency is thought to be the most common cause of anaemia globally, although other conditions, such as folate, vitamin B12 and vitamin A deficiencies, chronic inflammation, parasitic infections, and inherited disorders can all cause anaemia.
In its severe form, it is associated with fatigue, weakness, dizziness and drowsiness. Pregnant women and children are particularly vulnerable.
Studies from across the globe have shown that severe anaemia in mothers and premature delivery can also lead to childhood anaemia.

16.Solution: d)
AOSIS is a coalition of 44 small island and low-lying coastal developing states, including five observers. As a voice for the vulnerable, its mandate is more than amplifying marginalised voices as it also advocates for these countries’ interests. In terms of size, AOSIS closely resembles the countries it represents on the global stage, but often punches far above its weight, negotiating historic global commitments to cut greenhouse gas emissions, among other achievements.

17.Solution: c)
Sustainable Alternative Towards Affordable Transportation (SATAT) is an initiative aimed at setting up of Compressed Bio-Gas production plants and make it available in the market for use in automotive fuels by inviting Expression of Interest from potential entrepreneurs.
The initiative was launched in October 2018 by the Ministry of Petroleum & Natural Gas in association with Public Sector Undertaking (PSU) Oil Marketing Companies (OMC)

18.Solution: c)
European Union (EU) has become the first multilateral bloc to declare climate emergency.
Similar climate emergency declarations have already been made in several EU member states, including Spain, France and the United Kingdom. Outside Europe Canada, Argentina and Bangladesh have declared a climate emergency.

19.Solution: c)
Launched in 2000, Unesco Asia-Pacific awards for cultural heritage conservation programme is aimed at acknowledging the efforts taken to restore and conserve historical structures without affecting their heritage value in the region comprising 48 countries.

20.Solution: d)
Carbon offsetting allows a country to help reach its own emissions reduction targets by funding emission reductions in another country. Companies are also increasingly using carbon credits to offset their emissions.
The first major offsetting scheme, the U.N.s clean development mechanism (CDM), was set up under the 1997 Kyoto Protocol, in which 190 countries agreed country-by-country emission reduction targets.

21.Solution: c)
Chinese President Xi Jinping and his Russian counterpart Vladimir Putin inaugurated the “Power of Siberia” gas pipeline — a massive cross-border undertaking not only central to China’s energy security but also for bolstering special ties between Beijing and Moscow.
Russia has been a primary gas supplier to Europe, but the Power of Siberia is the first cross-border gas pipeline between Russia and China, adding a prominent eastern dimension to Moscow’s energy blueprint.
From Siberia to China’s Yangtze River delta in Shanghai, the massive pipeline will cover 8,000 km, with 5,111 km inside China, passing through nine provinces and municipalities.
Gas is being sourced from Chayandinskoye and Kovytka fields in eastern Siberia, and is then piped to Blagoveshchensk — the last town on the Russian side of the border. From there, it is tunneled under the Amur River, before entering Heihe on the Chinese side.

22.Solution: c)
YuWaah’- Generation Unlimited initiative:
Launched by UNICEF.
The target age group of YuWaah includes adolescent girls and boys and its key mission is to promote access to foundational, transferable and 21stcentury skills for youth inside and outside formal education systems, which includes defining foundational skills, life skills and flexible learning and identifying and scaling impactful delivery models.
YuWaah intends to create platforms to guide youth to market opportunities (career guidance, mentorship, internships, apprenticeships) and facilitate integration of career guidance in school education.
It brings young people together with the private sector, governments, international and local organisations.
The aim is to tackle the urgent challenge of investing in their learning and training so that they are prepared for the complex and fast-changing world of work and can be active and engaged citizens.

23.Solution: b)
‘Global Migration Report 2020’ has been released by International Organisation for Migration (IOM).
Key findings- India specific:
India accounts for the highest share with 17.5 million Indians living outside the country.
India is the leading recipient of remittances. International remittances in 2018 (2020 report) reached $689 billion, out of which India received $78.6 billion from the 17.5 million living abroad.
Remittances received by India have consistently increased between the 2005 and 2020.
The top migration corridors for Indians are the United Arab Emirates, the US and Saudi Arabia.
Highest number of migrants entering India come from Bangladesh.

24.Solution: a)
The Treaty on the Prohibition of Nuclear Weapons (TPNW), or the Nuclear Weapon Ban Treaty, is the first legally binding international agreement to comprehensively prohibit nuclear weapons, with the goal of leading towards their total elimination. It was passed on 7 July 2017. In order to come into effect, signature and ratification by at least 50 countries is required. For those nations that are party to it, the treaty prohibits the development, testing, production, stockpiling, stationing, transfer, use and threat of use of nuclear weapons, as well as assistance and encouragement to the prohibited activities. For nuclear armed states joining the treaty, it provides for a time-bound framework for negotiations leading to the verified and irreversible elimination of its nuclear weapons programme.

25.Solution: b)
Bharat Bond Exchange Traded Fund (ETF), the country’s first corporate bond exchange-traded fund aimed at retail investors, saw a strong response, with the issue getting subscribed around 1.7 times.
The issue saw wide participation across categories including individuals, non-resident Indians, high networth individuals, top tier corporates and foreign portfolio investors (FPIs).
The exchange traded fund will invest only in AAA-rated bonds of public sector companies.

26.Solution (d)
The National Investigation Agency (NIA) is a state agency established by the Indian Government to combat terror in India. It acts as the Central Counter Terrorism Law Enforcement Agency. The agency is empowered to deal with terror related crimes across states without special permission from the states. The Agency came into existence with the enactment of the National Investigation Agency Act 2008 by the Parliament of India on 31 December 2008.
It is under the aegis of the Ministry of Home Affairs.

27.Solution (a)
It is found in parts of Pakistan in the Indus river system of Kashmir and northern and central India along the Ganges, Bangladesh and Burma and formerly occurred in Laos, Cambodia and Vietnam. It is a scarce non-breeding visitor to Nepal and has occurred as a vagrant in Oman and central Thailand with old records from Iran and China.
IUCN – Vulnerable.

28.Solution (b)
The National Pharmaceutical Pricing Authority (NPPA) is a government regulatory agency that controls the prices of pharmaceutical drugs in India.National Pharmaceutical Pricing Authority (NPPA) was constituted vide Government of India Resolution dated 29th August, 1997 as an attached office of the Department of Pharmaceuticals (DoP), Ministry of Chemicals & Fertilizers as an independent Regulator for pricing of drugs and to ensure availability and accessibility of medicines at affordable prices.
Functions
To implement and enforce the provisions of the Drugs (Prices Control) Order in accordance with the powers delegated to it.
To deal with all legal matters arising out of the decisions of the Authority.
To monitor the availability of drugs, identify shortages, if any, and to take remedial steps.
To collect/ maintain data on production, exports and imports, market share of individual companies, profitability of companies etc, for bulk drugs and formulations.
To undertake and/ or sponsor relevant studies in respect of pricing of drugs/ pharmaceuticals.
To recruit/ appoint the officers and other staff members of the Authority, as per rules and procedures laid down by the Government.
To render advice to the Central Government on changes/ revisions in the drug policy.
To render assistance to the Central Government in the parliamentary matters relating to the drug pricing

29.Solution (a)
2I/Borisov, originally designated C/2019 Q4 (Borisov), is the first observed interstellar comet and the second observed interstellar interloper after ʻOumuamua.
It was observed by the NASA’s Hubble Space Telescope.


30.Q.5) Solution (b)
The OSIRIS-REx (Origins, Spectral Interpretation, Resource Identification, Security, Regolith Explorer) is a NASA asteroid study and sample-return mission. The mission’s main goal is to obtain a sample of at least 60 grams from 101955 Bennu, a carbonaceous near-Earth asteroid, and return the sample to Earth for a detailed analysis.

31.Solution (c)
Led by Chile, the Climate Ambition Alliance was launched at the Climate Action Summit in New York
‘Climate Ambition Alliance’ aims to accelerate the transformation needed to meet the goals of the Paris Agreement on climate change and stabilize the global average temperature rise at 1.5°C above preindustrial levels.
For mitigation, the Climate Ambition Alliance will focus on the submission of enhanced Nationally Determined Contributions; reaching new commitments to achieve Net Zero by 2050; and the implementation of measures to strengthen the protection of forests and oceans.
For adaptation, the Climate Ambition Alliance, will focus on strong actions to improve the management of water, resilience in infrastructure and the sustainability of cities. 

32.Solution (d)
Odisha Liveable Habitat Mission “JAGA” is a society under Housing & Urban Development Department, Government of Odisha, headed by the Chief Secretary, Odisha as Chairman and Principal Secretary, H&UD as Member Secretary. Odisha Liveable Habitat Mission (OLHM) – “JAGA” aims at transforming the slums into liveable habitat with all necessary civic infrastructure and services at par with the better off areas within the same urban local body (ULB) and to continuously improve the standard of the infrastructure and services and access to livelihood opportunities. This Mission also aims at leveraging and converging various schemes/ programs/ funding opportunities by strengthening collaboration among various Departments/ Urban Bodies/ Non-Government Organisations/ Financial Institutions/ International Agencies/ Trusts/ Communities and other Stakeholders. It will also provide advisory support to Government of Odisha to examine options for policy reforms required for the sustainable transformation of lives of urban poor.

33.Solution (b)
In July a new “East Mediterranean Gas Forum” (EMGF) was launched by those three countries, plus Egypt, Jordan, Italy and the Palestinian Authority. However, Turkey was excluded.
The Eastern Mediterranean is reckoned to be rich in natural gas: the US Geological Survey estimates the deposits to run into trillions of cubic metres, worth hundreds of billions of dollars, besides millions of barrels of oil.


34.Solution (a)

35.Solution (a)
India has moved down the ladder from its 108th position last year on the World Economic Forum’s Gender Gap Report.

36.Solution (a)
The Hong Kong Convention) is aimed at ensuring that ships, when being recycled after reaching the end of their operational lives, do not pose any unnecessary risk to human health and safety or to the environment.

37.Solution (c)
BrahMos is a joint venture of the DRDO of India and the NPOM of Russia.
It is a medium-range ramjet supersonic cruise missile that can be launched from submarine, ships, aircraft, or land. It is the fastest supersonic cruise missile in the world.
The missile is operational with the Indian Army, Navy and Air Force.

38. Solution (a)
Guided by the Global Compact on Refugees, the Global Refugee Forum is an opportunity to translate the principle of international responsibility-sharing into concrete action. The Forum will showcase impactful pledges and contributions and the exchange of good practices.
The first Global Refugee Forum, took place on 17 and 18 December 2019 at the Palais des Nations in Geneva, Switzerland, and focused on the following areas: arrangements for responsibility sharing, education, jobs and livelihoods, energy and infrastructure, solutions and protection capacity.
It is jointly hosted by the United Nations High Commissioner for Refugees (UNHCR), the UN Refugee Agency, and the government of Switzerland, aims to debate and discuss the response of the world’s countries to the global refugee situation.

39.Solution (c)
African swine fever (ASF) is a highly contagious haemorrhagic viral disease of domestic and wild pigs, which is responsible for serious economic and production losses.
It is caused by a large DNA virus of the Asfarviridae family, which also infects ticks of the genus Ornithodoros.
Although signs of ASF and classical swine fever (CSF) may be similar, the ASF virus is unrelated to the CSF virus.
ASF is a disease listed in the World Organisation for Animal Health (OIE) Terrestrial Animal Health Code and must be reported to the OIE.
ASF is not a risk to human health.

40.Solution (a)

41.Solution (a)
The scheme was launched in 2000. It was completely funded by the central government. In 2015, under the recommendation of the 14th finance commission, the project was funded by both centre and state in the ratio 60:40.
It is under the Ministry of Rural Development.

42.Solution (a)
HD 86081 b or Santamasa, meaning ‘clouded’ in Sanskrit, is a gas giant exoplanet that orbits close to its host star, completing its orbit for only 2.1375 days. With such a short orbit, it belongs to the class of exoplanets known as hot Jupiters. Like most hot Jupiters, the orbit is nearly circular, with an eccentricity of 0.008.
Source: http://vigyanprasar.gov.in/isw/A-star-and-its-planet-gets-Indian-names.html

43.Solution (b)
It aims to connect major districts and towns with semi high-speed trains that will run on their own tracks. The 532-km corridor is projected to be built at a cost of Rs 56,443 crore. Trains would complete the journey at four hours instead of 12, with a maximum speed of 200 km/h.

44.Solution (b)
Globally, more two-thirds of all refugees come from five countries: Syria (6.7 million), Afghanistan (2.7 million), South Sudan (2.3 million), Myanmar (1.1 million), and Somalia (0.9 million).
Countries in the developed regions host 16 per cent of refugees; one-third of the refugee population (6.7 million people) are in the Least Developed Countries.
The largest host countries are Turkey (3.7 million), Pakistan (1.4 million), Uganda (1.2 million), Sudan (1.1 million), and Germany (1.1 million).

45.Solution (b)
WHO launched a biosimilar medicine “Trastuzumab” in order to treat breast cancers. It is an antibody that shows high efficacy in curing early stages of breast cancers. It was included in the WHO Essential Medicines List in 2015.

46.Solution (a)
Pinaka is a multiple rocket launcher produced in India and developed by the Defence Research and Development Organisation (DRDO) for the Indian Army. The system has a maximum range of 40 km for Mark-I and 75 km for Mark-II, and can fire a salvo of 12 HE rockets in 44 seconds.

47.Solution (b)
The 2019 Pollution and Health Metrics: Global, Regional and Country Analysis report from the Global Alliance on Health and Pollution (GAHP) updates findings from The Lancet Commission on Pollution and Health, and provides a ranking of pollution deaths on global, regional and country levels. The report uses the most recent Global Burden of Disease data from the Institute of Health Metrics Evaluation.
India had most deaths caused by pollution in 2017. The report includes three lists on pollution-induced deaths. India is the only country that features in the top 10 in all three lists (right).

48.Solution (d)
India and Bangladesh share 54 rivers, but there is a limited water sharing treaty only on one – the Ganga.
Some of the major transboundary rivers between the countries are Ganga, Brahmaputra, Teesta and Surma-Kushiara.

49. Solution (b)
Exercise ‘Apharan’ is a large-scale Anti Hijacking Exercise conducted recently by the Indian Navy in collaboration with Indian Coast Guard. This year it was conducted in Kerala.

50.Solution (d)
IUCN – Vulnerable
MacQueen’s bustard (Chlamydotis macqueenii) is a large bird in the bustard family. It was earlier included as a subspecies of the houbara bustard (Chlamydotis undulata) and sometimes known as the “Asian houbara”.
The population of the Asian houbara bustards extends from northeast Asia, across central Asia, the Middle East, and the Arabian Peninsula to reach the Sinai desert.

51.Answer: a
Explanation:
CISF was set up under an Act of the Parliament of India on 10 March 1969. CISF was subsequently made an armed force of the Republic of India by another Act of THE Parliament passed on 15 June 1983.
CISF is directly under the Union Ministry of Home Affairs and not the Ministry of Defence.
The CISF provides security cover to 300 industrial units, government infrastructure projects and facilities and establishments located all over India.
Industrial sectors like atomic power plants, space installations, mines, oil fields and refineries, major ports, heavy engineering, steel plants, barrages, fertiliser units, airports and hydroelectric/thermal power plants owned and controlled by Central Public Sector Undertakings (PSUs), and currency note presses producing Indian currency are protected by CISF.
CISF also provides consultancy services to private industries as well as other organisations within the Indian government.

52.Answer: b
Explanation:
The Arihant-class submarines are nuclear powered ballistic missile submarines built under the Advanced Technology Vessel (ATV) project. They will be the first nuclear submarines designed and built by India.
INS Arihant is the lead ship of India’s Arihant class of nuclear-powered ballistic missile submarines. The vessel is classified as a Strategic Strike Nuclear Submarine by India. The submarines are powered by a pressurised water reactor with highly enriched uranium fuel. The Arihant class is armed with nuclear capable missiles and is critical to India’s second strike capability.
INS Arighat is the second Arihant-class submarine. It is the second nuclear-powered ballistic missile submarine being built by India.
INS Varsha is a new naval base being developed under Project Varsha for the Indian Navy. This base will be the home of the navy’s new fleet of nuclear submarines and ships. It was planned to be located within a radius of approximately 200 kilometres (124.27 statute miles) from Visakhapatnam, the headquarters of the navy’s Eastern Naval Command.
The INS Arihant and Chakra – on lease from Russia – are the two nuclear-powered submarines currently in service with the Navy.
Sagarika also known by the code names K-15 or B-05, is a nuclear-capable submarine-launched ballistic missile with a range of 750 kilometres (466 mi). It belongs to the K Missile family and forms a part of India’s nuclear triad, and will provide retaliatory nuclear strike capability.

53.Answer: d
Juvenile Justice (Care and Protection of Children) Act, 2015 replaces the Indian juvenile delinquency law, Juvenile Justice (Care and Protection of Children) Act, 2000, and allows for juveniles in conflict with Law in the age group of 16–18, involved in Heinous Offences, to be tried as adults. The crime will be examined by the Juvenile Justice Board to ascertain if the crime was committed as a ‘child’ or an ‘adult’.
The Act also sought to create a universally accessible adoption law for India, overtaking the Hindu Adoptions and Maintenance Act (1956) (applicable to Hindus, Buddhists, Jains, and Sikhs) and the Guardians and Wards Act (1890) (applicable to Muslims), though not replacing them.
A separate chapter on Adoption provides detailed provisions relating to adoption and punishments for non-compliance. Processes have been streamlined with timelines for both in-country and inter-country adoption including declaring a child legally free for adoption.
To streamline adoption procedures for orphan, abandoned and surrendered children, the existing Central Adoption Resource Authority (CARA) has been given the status of a statutory body to enable it to perform its function more effectively.

54.Answer: c
The Foreign reserves of India consists of below four categories (in decreasing proportion):
Foreign Currency Assets
Gold (The RBI holds around 618 tonnes of gold, of which around 325 tonnes are held abroad with the Bank of England and the Bank for International Settlements.)
Reserve Tranche Position with IMF
Special Drawing Rights (SDRs) with IMF

55.Answer: b
Explanation:
The National Population Register (NPR) is a Register of usual residents of the country.
The objective of the NPR is to create a comprehensive identity database of every usual resident in the country. The database would contain demographic as well as biometric particulars.
It is being prepared at the local (Village/sub-Town), sub-District, District, State and National level under provisions of the Citizenship Act 1955 and the Citizenship (Registration of Citizens and Issue of National Identity Cards) Rules, 2003.
It is mandatory for every usual resident of India to register in the NPR.
Definition: A usual resident is defined for the purposes of NPR as a person who has resided in a local area for the past 6 months or more or a person who intends to reside in that area for the next 6 months or more.
As per the provisions of the NPR, a resident identity card (RIC) will be issued to individuals over the age of 18. This will be a chip-embedded smart card containing the demographic and biometric attributes of each individual. The UID number will also be printed on the card.

56.Answer: a
Explanation:
Operation Twist is the name given to the monetary policy operation that involves the purchase and sale of bonds. On a review of the current liquidity and market situation and an assessment of the evolving financial conditions, the Reserve Bank has decided to conduct simultaneous purchase and sale of government securities under Open Market Operations (OMO) for Rs. 10,000 crore each.

57.Answer: c
Explanation:
COMCASA is one of the four foundational agreements that guide US high technology cooperation in the defence sector with other countries. COMCASA is meant to facilitate the use of high-end secured communication equipment to be installed on military platforms being sold to India by the US to fully exploit their potential. It essentially provides a legal framework for the transfer of communication security equipment from the US to India that will facilitate interoperability between armed forces of both countries and potentially with other militaries that use US-origin systems for secure data links. Interoperability, in this case, means that there will be access to encrypted and secret technologies or communications.

58.Answer: c
Explanation:
INDRA 2019 is a joint tri-services exercise between India and Russia. The INDRA series of exercise began in 2003. It was conducted as a single service exercise alternately between the two countries. However, the first joint Tri-Services Exercise was conducted in 2017.

59.Answer: b
Explanation:
There is no maximum time-limit within which a mercy petition has to be decided.
The mercy petition is subject to Judicial Review.
In India, the authority to decide on mercy petitions rests with the President and the Governor.

60.Answer: b
Explanation:
There are more than 300 rivers in Bangladesh of which 57 are trans-boundary rivers. Out of the 57 transboundary rivers, 54 are common with India and remaining 3 with Myanmar. The country is surrounded on three sides by India. It has a gigantic delta formed by the alluvial deposits of the three rivers: the Ganges, the Brahmaputra and the Meghna.

61.Answer: d

62.Answer: d
Explanation:
National Company Law Appellate Tribunal (NCLAT) was constituted under the Companies Act, 2013.
It hears appeals against the orders of National Company Law Tribunal(s) (NCLT), with effect from 1st June 2016.
It is the Appellate Tribunal for hearing appeals against the orders passed by NCLT(s) under Section 61 of the Insolvency and Bankruptcy Code, 2016 (IBC).
It is the Appellate Tribunal for hearing appeals against the orders passed by Insolvency and Bankruptcy Board of India under Section 202 and Section 211 of IBC.
It is the Appellate Tribunal to hear and dispose of appeals against any direction issued or decision made or order passed by the Competition Commission of India (CCI).

63.Answer: b
Explanation:
A solar eclipse occurs on a new moon day when the Moon comes in between the Earth and the Sun and when all the three celestial bodies are aligned. A solar eclipse is a phenomenon that happens when the moon comes in the way of the sun’s light. The moon’s shadow casts itself on Earth, blocking out the sun’s light (as seen from Earth).

64.Answer: a
Explanation:
Red Octagon is a warning symbol employed in packaged foods in Chile and Peru. It has a number and the name of the food component within that indicates how widely off the RDA a particular ingredient is. Centre for Science and Environment (CSE) has concluded that, in India, all of the popular snacks and fast foods should display a ‘Red Octagon’ on the front of the pack.

65.Answer: b
Explanation:
Maternal mortality refers to deaths due to complications from pregnancy or childbirth. Maternal mortality ratio is calculated per 100,000 live births. India has seen a declining trend of Maternal Mortality Rate since the year 2007.
In India, the Sample Registration Survey (SRS) is used to get an estimate of the maternal mortality rate. Given below is the MMR in India as per the Sample Registration System:
Maternal Mortality Rate (MMR) in India
Year          MMR
2004-2006           254
2007-2009           212
2010-2012           178
2011-2013           167
2014-2016           130
2015-2017           122

66.Answer: c
Explanation:
The Missile Technology Control Regime (MTCR) is a multilateral export control regime. It is an informal political understanding among 35 member states that seeks to limit the proliferation of missiles and missile technology. India has become the 35th member of the Missile Technology Control Regime (MTCR) in June 2016. The MTCR membership will enable India to buy high-end missile technology.

67.Answer: b
Explanation:
The Act gives eligibility for Indian citizenship to illegal migrants who are Hindus, Sikhs, Buddhists, Jains, Parsis and Christians from Afghanistan, Bangladesh and Pakistan. The Act does not apply to the tribal areas of Assam, Meghalaya, Tripura, Arunachal Pradesh, Mizoram and Nagaland.

68.Answer: c
Explanation:
Light Detection and Ranging is a remote sensing method that uses light in the form of a pulsed laser to measure variable distances to the Earth. LiDAR can be used for monitoring pollution. Topographic LiDAR typically uses a near-infrared laser to map the land, while bathymetric LiDAR uses water-penetrating green light to also measure seafloor and riverbed elevation.

69.Answer: d
Explanation:
Export Credit Guarantee Corporation of India (ECGC) introduced ‘NIRVIK’ scheme to ease the lending process and enhance loan availability for exporters. Under the new ‘NIRVIK’ scheme, which is also called the Export Credit Insurance Scheme (ECIS), the insurance cover guaranteed will cover up to 90 percent of the principal and interest. The insurance cover will include both pre and post-shipment credit.

70.Answer: b
Explanation:
Exercise SURYA KIRAN is a bilateral annual military exercise. The joint military training exercise is held between India and Nepal.

71.Answer: a
Explanation:
Mukna is a form of folk wrestling from the north-east Indian state of Manipur. It is popular in Imphal, Thoubal and Bishnupur. The game is generally played on the last day of the Lai Haraoba festival and is an intrinsic part of the ceremonial functions.
Huyen Langlon is an Indian martial art from Manipur. In the Meitei language, huyen means war while langlon or langlong can mean net, knowledge or art. Huyen Langlon consists of two main components: thang-ta (armed combat) and sarit sarak (unarmed fighting). The primary weapons of Huyen Langlon are thang (sword) and ta (spear).
Silambam is a weapon-based martial art of India, more specifically from Tamilakam in the Indian subcontinent.
Paika akhada is a martial art found in Odisha.

72.Answer: c
Explanation:
Part XI of the Constitution, which governs the relationship between the Centre and the states, says categorically that states are bound to implement the laws passed by Parliament.
Article 256 of the Constitution makes it a state’s obligation to ensure compliance with central laws.
Article 249 empowers Parliament to make laws even on state subjects in the national interest.
Under Articles 251 and 254, the Centre is to prevail over the states in the event of any inconsistency between central and state legislations.
The governor can recommend President’s rule under Article 356, advising that the state cannot be run in keeping with the Constitution’s provisions.

73.Answer: d
Explanation:
Toda embroidery is associated with Tamil Nadu. The Toda embroidery, also locally known as “pukhoor”, is an artwork among the Toda pastoral people of Nilgiris in Tamil Nadu. It is made exclusively by women.

74.Answer: d
Explanation:
The Maternity Benefit Act, 1961 protects the employment of women during the time of her maternity and entitles her of a ‘maternity benefit’ – i.e. full paid absence from work – to take care for her child. The act is applicable to all establishments employing 10 or more employees.
The Maternity Benefit Amendment Act has increased the duration of paid maternity leave available for women employees from the existing 12 weeks to 26 weeks.

75.C. Both 1 and 2
Explanation :
Recently, Minister of Shipping informed Lok Sabha that the Government has taken many steps to clear the uncertainty of the Shipping industry arising out of impending IMO regulations to reduce the level of sulphur oxide emissions from ship's exhaust from January, 2020. International Maritime Organization (IMO) regulations to reduce sulphur oxides (SOx) emissions from ships first came into force in 2005, under Annex VI of the International Convention for the Prevention of Pollution from Ships (known as the MARPOL Convention). Since then, the limits on sulphur oxides have been progressively tightened. From 1 January 2020, the limit for sulphur in fuel oil used on board ships operating outside designated emission control areas will be reduced to 0.50% m/m (mass by mass). This will significantly reduce the amount of sulphur oxides emanating from ships and should have major health and environmental benefits for the world, particularly for populations living close to ports and coasts. Hence both statements are correct.

76.A.1 only
Explanation :
Statement 1 is correct:
The Supreme Court has dismissed a plea challenging a Kerala High Court order which held that an educational institution to claim minority status has be “established and administered” by the minority community, and not merely administered by it. The bench observed during the hearing that it was clear that under Article 30(1), for an institution to claim minority status, it has to be both "established and administered" by the community. Article 30(1) of the Constitution of India gives linguistic and religious minorities a fundamental right to establish and administer educational institutions of their choice.
Statement 2 is incorrect:
The Supreme Court in T.M.A. Pai Foundation vs. State of Karnataka (2002) held that a minority, whether linguistic or religious, is determinable only by reference to demography of the State and not by taking into consideration the population of the country as a whole. The National Commission for Minority Educational Institutions Act has been enacted to safeguard the educational rights of the minorities enshrined in Article 30(1) of the Constitution. Section 2(g) of the Act defines a Minority Educational Institution as a college or institution (other than a University) established or maintained by a person or group of persons from amongst the minorities.

77.A.1 only
Explanation :
Statement 1 is correct:
International climate talks at COP25 closed with no deal on carbon markets. The negotiators postponed until next year a key decision on global carbon markets.
Statement 2 is incorrect:
The 2019 United Nations Climate Change Conference, also known as COP25, is the 25th United Nations Climate Change conference. It was held in Madrid, Spain, from 2 to 13 December 2019 under the presidency of the Chilean government. The conference incorporates the 25th Conference of the Parties to the United Nations Framework Convention on Climate Change (UNFCCC), the 15th meeting of the parties for the Kyoto Protocol (CMP15), and the second meeting of the parties for the Paris Agreement (CMA2). The final declaration underscored the “urgent need” to cut planet-heating greenhouse gases in line with the goals of the landmark 2015 Paris climate change accord. That fell far short of promising to enhance countries’ pledges to cut greenhouse gases next year which developing countries had lobbied the delegates to achieve. Negotiators left some of the thorniest issues for the next climate summit in Glasgow in a year, including the liability for damages caused by rising temperatures that developing countries were insisting on.

78.C. Both 1 and 2
Explanation :
Coal Ministry has decided to establish a Sustainable Development Cell (SDC) to promote environmentally sustainable coal mining in the country. The Cell will advise, mentor, plan and monitor the mitigation measures taken by the coal companies for maximizing the utilization of available resources in a sustainable way, minimizing the adverse impact of mining and mitigating it for further eco-system services. It will address environmental concerns during the decommissioning or closure of mines. The move gains significance as the new private entities are now going to form a significant part of the future. Hence both statements are correct.

79.B. 2 only
Explanation :
In pursuance to objective of providing greater “ Ease of Doing Business”, the Ministry of Corporate Affairs (MCA) has taken several landmark initiatives / decisions during last one year ( January-November, 2019).
These are:
Simplified Proforma for Incorporating Company Electronically (SPICe) introduced which extends 8 services (CIN, PAN, TIN, DIN, Name, EPFO, ESIC and GSTN) from three Ministries through a single form.
De-criminalization of technical & procedural violations under Companies Act and reducing the burden on criminal courts & NCLT by shifting 16 offences sections to monetary penalty regime vide Companies (Amendment )Bill , 2019 notified on 31st July ,2019
Government Process Re-Engineering by Introducing “RUN – Reserve Unique Name” web service for name reservation for companies & LLP and Re-engineering the Process of allotment of Director Identification Number (DIN).
Provisions relating to issue of shares with Differential Voting Rights (DVRs) modified with the objective of enabling promoters of Indian companies to retain control of their companies, even as they raise equity capital from global investors.
Enabling provisions with regard to Mediation and Conciliation under the Companies Act, 2013 enforced.
Provisions relating to creation of Debenture Redemption Reserve (DRR) revised with the objective of deepening the bond market & reducing the cost of capital.
Independent Director’s Databank launched to provide an easy to access & navigate platform for the registration of existing Independent Directors as well as individuals aspiring to become independent directors.
Setting up Central Registration Centre for name reservation and incorporation of companies & LLPs within 1-2 days as opposed to an average of at least 15 days earlier.
First National CSR Awards distributed
Hence only statement 2 is correct.

80.A. Bug in android applications
Explanation :
The Union Home Ministry has sent an alert to all States warning them about the vulnerability of the Android operating system to a bug called ‘StrandHogg’. StrandHogg allows real-time malware applications to pose as genuine applications and access user data of all kind. These malware can then potentially listen to their conversations, access photo album, read/send messages, make calls, record conversations and get login credentials to various accounts. While all versions of Android are vulnerable to this bug, it may not be apparent to the affected users that malware applications are already on board their device. Pop-ups asking for permission to send notifications, messages etc., are one of the main entry points for ‘StrandHogg’ to launch the attack. Hence, option (a) is the correct answer.

81.C. Both 1 and 2
Explanation :
Supreme Court has asked government to look into the feasibility of hydrogen-based tech to deal with vehicular air pollution in capital. India is looking closely at Japan, which is gearing up to put on its roads thousands of vehicles based on a hydrogen cell technology, also known as ‘fuel cells’. At the heart of the fuel cell electric vehicles (FCEV) is a device that uses a source of fuel, such as hydrogen, and an oxidant to create electricity by an electrochemical process. The fuel cell combines hydrogen and oxygen to generate an electric current, water being the only byproduct. Like conventional batteries under the bonnets of automobiles, hydrogen fuel cells too convert chemical energy into electrical energy.
Advantages of fuel cells: Fuel cells produce much smaller quantities of greenhouse gases and none of the air pollutants that cause health problems. Such cells are also far more energy efficient than traditional combustion technologies. Unlike battery-powered electric vehicles, fuel cell vehicles do not need to be plugged in.
Disadvantages of fuel cells: The process of making hydrogen needs energy which is often from fossil fuel sources. That has raised questions over hydrogen’s green credentials. There are questions of safety as hydrogen is more explosive than petrol. The vehicles are expensive, and fuel dispensing pumps are scarce.
Hence both statements are correct.

82.A.1 only
Explanation :
The Government is testing a prototype of an Indian equivalent of popular messaging platforms, such as WhatsApp and Telegram, for secure internal use. Government Instant Messaging System (GIMS) is being packaged for employees of Central and state government departments and organisations for intra and inter organisation communications. It is being developed as a secure Indian alternative without the security concerns attached with apps hosted abroad or those owned by foreign entities. Like WhatsApp, GIMS employs end-to-end encryption for one-to-one messaging. GIMS is being touted as a safer bet as the platform has been developed in India, the server hosting it is installed within the country and the information stored would be in government-based cloud — NIC-operated data centres that are only meant for captive use by the government and its departments. It is designed and developed by the Kerala unit of National Informatics Centre (NIC). The platform is in the pilot testing stage across some states, including Odisha — and is learnt to have been released to the Indian Navy to be tried out on trial basis. Hence only statement 1 is correct.

83.A. New Zealand
Explanation :
As the toll in the New Zealand volcano disaster climbed to 16, some Maori feel this volcano disaster was a form of divine retribution. The Māori are the indigenous Polynesian people of New Zealand. In the 2018 census, there were 775,836 people in New Zealand identifying as Māori, making up 16.5 per cent of the national population. In addition, more than 140,000 Māori live in Australia. White Island is a jewel of Aotearoa — the Maori name for New Zealand — and the local Ngati Awa tribe consider Whakaari to be a living ancestor. In the Maori cognition, it is wrong for anyone to stomp on her sacred soil. Hence, option (a) is the correct answer.

84.A.1 only
Explanation :
The Railway Ministry clearing the decks for the Integral Coach Factory (ICF) here to continue making Vande Bharat Express trains is seen as a welcome move though senior officials are sceptical of rolling out 45 rakes, 720 coaches, by 2021-22. Integral Coach Factory (ICF) is a manufacturer of rail coaches located in the neighbourhood of Perambur, Chennai, Tamil Nadu. It was established in 1952 and was inaugurated by Jawaharlal Nehru in 1955. It is owned and operated by the Indian Railways. The ICF is one of the three rake production units of the Indian Railways, the other two being the Modern Coach Factory at Raebareli and the Rail Coach Factory at Kapurthala. The new age flagship Trainset code-named Train 18 was rolled out by ICF by the end of October, 2018. Hence only statement 1 is correct.

85.Answer : d
Recently, Quality Council of India (QCI) along with PHD Chamber of Commerce and Industry (PHDCCI)organized 11thRegional Quality Conclave (RQC).
The theme of theRQC is ‘Advance Manufacturing with Quality, Innovation & Technology Interventions.’
The Conclave aims to create awareness about the significance of quality culture in businesses.
QCI is an autonomous organization under the Department for Promotion of Industry and Internal Trade(DPIIT), Ministry of Commerce and Industry.

86.Answer : a
Human Solidarity Day or International Human Solidarity Dayis annually celebrating on 20th December by UN.
The General Assembly, on 22 December 2005, by “Resolution 60/209”,identified solidarity as one of the fundamental and universal values that should underlie relations between peoples in the 21stcentury, and in that regard decided to proclaim 20thDecember of each year International Human Solidarity Day.
By Resolution 57/265 the General Assembly, established the World Solidarity Fund, its objective is to eradicate poverty and promote human and social development in developing countries, in particular among the poorest segments of their populations.

87.Answer : d
The International Astronomical Union (IAU) has announced the names of newly discovered star as ‘Sharjah' and its planet'Barajeel'.
A barajeel is a wind tower used to direct the flow of the wind so that air can be recirculated as a form of air conditioning.

88.Answer : d
Chennai Corporationlaunched India’s 1stWaste Exchange platform namely “Madras Waste Exchange”.
The Madras Waste Exchange, is both a web portal and an application, has been conceptualised by the Smart City Mission, with support from the Union Ministry of Housing and Urban Affairs.

89.Answer : d
The European Union (EU)launched its flagship economic policy, the European Green Deal.
‘Climate neutral’ Europe,  this is the overarching objective of the European Green Deal.
The EU will aim to reach net-zero greenhouse gas emissions by 2050.
A new circular economy action plan will be tabled in March 2020, as part of a broader EU industrial strategy.
Building renovationis meant to be one of the flagship programmes of the Green Deal.
The key objective is to “at least double or even triple” the renovation rate of buildings, which currently stands at around 1%.
Zero-pollution:Whether in air, soil or water, the objective is to reach a “pollution-free environment” by 2050.

90.Answer : a
EChO Network is a national program launched by the Government of India (GoI) recently.
It aims to provide a template for cross-disciplinary leadership in India with the specific focus of increasing research, knowledge, and awareness of Indian ecology and the environment.
EChO Network would develop a national network to catalyse a new generation of Indians who can synthesize interdisciplinary concepts and tackle real-world problems in medicine, agriculture, ecology, and technology. With no precedent for such a network anywhere in the world, EChO Network establishes a new platform to change how science is embedded in our modern society.

91.Answer : b
The Competition Commission of India (CCI) has recently received three Green Channel combinations for automatic approval under the competition law.
Green Channel Clearance
The Competition Commission of India (CCI) introduced an automatic system of approval for combinations under 'Green Channel'.

92.Answer : b
The Indian Pharmacopoeia (IP) has been recently recognised formally by the National Department of Regulation of Medicines and Health Products of the Ministry of Public Health of Islamic Republic of Afghanistan.
It will also be used based on the requirement as reputable pharmacopoeia in the laboratory of medicines and health products quality.

93.Answer : a
NASA’s InSight touched down in November 2018 on a quest to understand the interior of the Mars – The Red Planet.
Two of its crucial tools for that task were a burrowing heat probe nicknamed the “mole” and a super-sensitive seismometer to study motion within the planet.

94.Answer : c
Macau, a tiny former Portuguese colony, is marking the 20th anniversary of its return to China.
Macau, special administrative region of China, on the country’s southern coast.
Macau is located on the south-western corner of the Pearl (Zhu) River (Chu Chiang) estuary (at the head of which is the port of Guangzhou) and stands opposite the Hong Kong Special Administrative Region, which is located in the eastern side of the estuary.

95.Answer : a
Union Defence Minister and External Affairs Minister met US Secretary of Defense and Secretary of State in Washington DC recently for the second annual India-US 2+2 Ministerial Dialogue.
This 2+2 Ministerial is the highest-level institutional mechanism between the two countries and provides for a review of the security, defence and strategic partnership between India and the United States.
This was the second 2+2 meeting and the first in US.

96.Answer : c
Upgraded version of Pinaka guided rocket system was successfully test-fired from base on Odisha coast recently.
Pinaka is an artillery missile system capable of striking into an enemy territory up to a range of 75 km with high precision.
The Pinaka Mk-II rocket is modified as a missile by integrating with the navigation, control and guidance system to improve the end accuracy and enhance the range.
The navigation system of the missile is also aided by the Indian Regional Navigation Satellite System (IRNSS) also called as NAVIC.

97.Answer : c
To enhance women safety in the country, Government has formulated a multi-pronged action plan and initiated a series of measures.
ITSSO is one of such measures developed by MHA towards Smart Policing.
The Investigation Tracking System for Sexual Offences (ITSSO) is an online module available to law enforcement agencies at all levels- National, State, District and Police Station.
It allows State to undertake real-time monitoring and management for completion of investigation in rape cases in 2 months.
 Image result for UPSC IMAGES
98.Answer : a
In a first, the Reserve Bank of India (RBI) announced a simultaneous sale and purchase of government bonds under the Open Market Operations mechanism, on lines of the “Operation Twist” that was last adopted by the US Federal Reserve in 2013.
Operation Twist is a move taken by U.S. Federal Reserve to make long-term borrowing cheaper.

99.Answer : a
Odisha has recently launched the 'Jalsathi' programme to ensure supply of safe drinking water to all households in the state.

100.       Answer : a
As protesters against the Citizenship Amendment Act hit the streets in large numbers in several states, state governments sought to tamp down on the demonstrations by issuing prohibitory orders under Section 144 of the Code Of Criminal Procedure (CrPC), 1973.
Section 144 CrPC, a law retained from the colonial era, empowers a district magistrate, a sub-divisional magistrate or any other executive magistrate specially empowered by the state government in this behalf to issue orders to prevent and address urgent cases of apprehended danger or nuisance.

101.       Answer : b
The New Foreign Trade Policy will come into effect by April 1, 2020 – office of the Directorate General of Foreign Trade (DGFT).
DGFT is an attached office of the Ministry of Commerce and Industry and is headed by Director General of Foreign Trade.
With the liberalization in the economic policies of the Government, DGFT has since been assigned the role of “facilitator”.
The shift was from prohibition and control of imports/exports to promotion and facilitation of exports/imports, keeping in view the interests of the country.

102.       Answer : d
Recently, the German government signed off a new law banning "Gay conversion therapies”
The idea of conversion therapies is linked to the one that sees homosexuality as a mental disorder.
This is a view that mental health professionals reject.
It purports to “cure” homosexuality by turning gay people, s

103.       Answer : c
Kambala (slush-track buffalo race) organisers are planning to establish a museum on this traditional sport.
It is an annual buffalo race conducted in Coastal belt of Karnataka.
It is a traditional bull sport similar to Jallikattu in Tamil Nadu.
The 'track' used for Kambala is a slushy paddy field.

104.       Answer : d
Uzbekistan has won the “Country of the year” award from ‘The Economist’.
The edition annually awards the states achieving the biggest progress in democracy.
The selection criteria include not the financial situation, influence, cuisine of the country, but the improvements the country demonstrated.
 Image result for UPSC IMAGES

No comments:

Post a Comment